LSAT and Law School Admissions Forum

Get expert LSAT preparation and law school admissions advice from PowerScore Test Preparation.

 Administrator
PowerScore Staff
  • PowerScore Staff
  • Posts: 8919
  • Joined: Feb 02, 2011
|
#72669
Complete Question Explanation

Main Point. The correct answer choice is (B).

The author argues that there must be a lake of liquid water between the rocky core and the icy surface of Enceladus, based on evidence provided by the results of the Cassini probe. The use of "there must be" in the second sentence should help us to realize that this is the author's conclusion. Everything after that in the stimulus is presented as irrefutable fact, and those are therefore the premises.

Answer choice (A): The author isn't trying to prove this statement, but presented it to us as an established fact as background information.

Answer choice (B): This is the correct answer choice. As indicated above, the author presents the evidence from the Cassini probe's measurements to support this claim, so this is the conclusion of the argument.

Answer choice (C): This is more factual background information, not a conclusion based on any evidence,

Answer choice (D): This is a premise, not supported by anything else but used to support the claim about their being a lake of liquid water between the layers on Enceladus.

Answer choice (E): This is one of the premises of the argument, not a conclusion. We know this because it was presented as a statement of fact, with no support offered for it.
 cargostud
  • Posts: 17
  • Joined: Dec 23, 2019
|
#73000
I picked (E) for this one. Very similar answer to (B). (E) provides evidence for the conclusion, but (B) is the best expression of the conclusion.
 James Finch
PowerScore Staff
  • PowerScore Staff
  • Posts: 943
  • Joined: Sep 06, 2017
|
#73011
Hi Cargo Stud,

(B) and (E) are actually quite different: this is a Main Point question, and is asking for a direct restatement/paraphrase of the conclusion of the stimulus. This means that the correct answer cannot be a restatement of any other part of the stimulus, especially not a premise nor an intermediate conclusion. Let's look at the answer choices:

(A)--Restates the background information/setup for the argument for the stimulus. Incorrect.

(B)--Restates the main conclusion, making it correct. Note the "there must be," which indicates an inference being made/conclusion being drawn.

(C)--More background information. Incorrect.

(D)--Serves as the affirmation of the sufficient condition in the conditional statement that immediately follows it. These two premises combine to allow the conclusion to be properly inferred. A premise, not the conclusion, hence incorrect.

(E)--The conditional statement that ends the stimulus. The preceding sentence tells us that the sufficient conditniona is true, allowing the necessary condition to be inferred as the stimulus's conclusion. A premise, not the conclusion, hence incorrect.

Hope this clears things up!

Get the most out of your LSAT Prep Plus subscription.

Analyze and track your performance with our Testing and Analytics Package.